subject
Mathematics, 22.04.2020 01:36 anthonysf

Using strong induction (so, neither weak nor "weak++") on the number of matches in the pile, prove that if this number is of the form 4k + 1, for k ∈ N, then P2 always has a strategy to win (which they will follow, because they are smart). Otherwise, P1 always have a strategy to win (which they will also follow because they’re smart too)! To be clear, you will need to prove both facts with a single inductive proof! That is to say, you will need to prove that P2 can always win under the aforementioned condition, but whenever that condition is not met, you have to prove that it is now P1 that can always win!

ansver
Answers: 2

Another question on Mathematics

question
Mathematics, 21.06.2019 12:40
Aparallelogram has two side lengths of 5 units. three of its sides have equations y = 0, y = 2, y = 2x. find the equation of the fourth side.
Answers: 1
question
Mathematics, 21.06.2019 16:30
What are “like terms”? why can we only add like terms?
Answers: 1
question
Mathematics, 21.06.2019 20:00
Frank owns a $141,000 home, for which he has a 30-year mortgage in the amount of $700 a month. once he has paid off mortgage, how much will he have paid in investment? a. $111,000 b. $109,000 c. $120,000 d. $141,000
Answers: 1
question
Mathematics, 21.06.2019 20:00
Anyone? 15m is what percent of 60m; 3m; 30m; 1.5 km?
Answers: 1
You know the right answer?
Using strong induction (so, neither weak nor "weak++") on the number of matches in the pile, prove t...
Questions
question
Mathematics, 06.10.2019 09:30